User avatar
 
ohthatpatrick
Thanks Received: 3808
Atticus Finch
Atticus Finch
 
Posts: 4661
Joined: April 01st, 2011
 
This post thanked 1 time.
 
 

Re: Q17 - Consumer: If you buy a watch

by ohthatpatrick Fri Dec 31, 1999 8:00 pm

Question Type:
Necessary Assumption

Stimulus Breakdown:
Conclusion: Store should give me a refund for the watch.
Evidence: even though BJ is not a dept store, if you buy a watch at dept store and only use it as intended and the watch stops the next day, you get a refund. The watch I bought at BJ's stopped working the next day.

Answer Anticipation:
Couple things missing here:
1. Why does BJ's need to follow a rule that dept stores follow, if BJ is not a dept store? The author is assuming that BJ's should follow at least some rules that apply to department stores.
2. According to the dept store policy, in order to give a refund, we need to know that you used the watch only as intended and it stopped working the next day. We know the watch stopped working the next day, but we don't know whether the customer used the watch only as it was intended.

Correct Answer:
D

Answer Choice Analysis:
(A) The author's argument is only concerned with whether or not he should get a refund, so this rule about "whether or not a store should sell something" is completely out of scope.

(B) Although the author needs to assume that BJ will adhere to at least one refund policy that dept stores do, he doesn't need to assume anything about BJ's watch quality being comparable to a dept store's.

(C) If you were using Sufficient Assumption brain and you assumed that "a watch that stops working the next day" = "a product that didn't perform as the purchaser expected", then you would like this answer, since it would guarantee you that the author should get a refund. But this is Necessary Assumption, and we can't accuse the author of assuming something stronger/broader than what he needs to assume. This conditional rule says that EVERY SINGLE TIME a product doesn't perform as expected, the seller should refund the money. The author is only concerned about whether he deserves a watch refund from BJ's. So he doesn't need to assume such a sweeping rule as this.

(D) Yes! If we negate this, we get "the customer DID use the watch in a way contrary to how it's intended to be used" … that cripples the argument because part of the refund policy the customer is citing involves that the buyer used the product "ONLY in the way it was intended".

(E) Nothing in the argument specifies new vs. used, so the author doesn't need to assume anything about that.

Takeaway/Pattern: There were two big gaps we could have anticipated: "Why should BJ's follow some dept store's rule?" and "Did the author use the watch only as intended?" They rewarded us with the latter. The trap answer of (C) probably tempts many people, if they aren't clear on the fact that we can't go overboard when selecting a Necessary Assumption answer.

#officialexplanation
 
perng.yan
Thanks Received: 2
Forum Guests
 
Posts: 51
Joined: November 05th, 2010
 
 
trophy
First Responder
 

Q17 - Consumer: If you buy a watch

by perng.yan Tue Dec 07, 2010 3:53 am

Again, my initial response was to choose (D)... because that seemed the logical/normal thing that most people would want to know...

however.. the part about "even though they themselves are not a department store" threw me off and I thought that the answer choice was supposed to show how Bringham's Jewelry and a department store are comparable so that this customer could make the claim that.. even though BJ and a department store are different, BJ SHOULD give a refund.

even if the customer used it like how it was intended.. BJ is not a department store and doesn't need to adhere to that rule.

Therefore I chose (B), because it links how BJ and the department store both sell watches that SHOULD last longer if they are used in the intended way.


Thanks again.
User avatar
 
bbirdwell
Thanks Received: 864
Atticus Finch
Atticus Finch
 
Posts: 803
Joined: April 16th, 2009
 
This post thanked 1 time.
 
 

Re: Q17 - Consumer: If you buy a watch

by bbirdwell Wed Dec 08, 2010 11:45 am

Here're the premises in a nutshell:

Get a watch at a department store, use it as it is intended to be used. If it stops working next day, get a refund.

BJ is not a department store.
Bought a watch there, it stopped working next day.

Conclusion: Should get a refund.

I see two gaps. One, as you said, is that BJ should act like a department store. The other is very important, though. If you look closely, you'll see that the department store example requires "intended use," and the evidence regarding BJ's never says that the consumer used it in the intended way.

It's as if the evidence says:
use as intended & stops working next day --> refund

and the conclusion says:
stops working next day --> refund

What's missing? What (D) says.

(B) is not correct because it doesn't matter how long the watches can be "expected" to work. We have no evidence as to how long either of the watches are expected to last, and that doesn't matter anyway. Maybe a department store watch is expected to last 10 years and BJ watch is expected to last 5. It doesn't affect our argument about "intended use" and next-day refunds.
I host free online workshop/Q&A sessions called Zen and the Art of LSAT. You can find upcoming dates here: http://www.manhattanlsat.com/zen-and-the-art.cfm
 
perng.yan
Thanks Received: 2
Forum Guests
 
Posts: 51
Joined: November 05th, 2010
 
 
trophy
First Responder
 

Re: PT57, S3, Q17 consumer: if you buy a watch

by perng.yan Fri Dec 10, 2010 4:49 pm

hm... it's hard not to get bogged down by useless info and thinking about things that don't really apply. I think I'll need to eliminate answer choices that like more aggressivley and stick to the strong and obvious ones like (D)

thanks for answering these nitpicky questions.
User avatar
 
bbirdwell
Thanks Received: 864
Atticus Finch
Atticus Finch
 
Posts: 803
Joined: April 16th, 2009
 
 
 

Re: PT57, S3, Q17 consumer: if you buy a watch

by bbirdwell Sat Dec 18, 2010 7:23 pm

No problem! You are absolutely right -- one of the biggest challenges for the whole test is learning to quickly identify what is useless information, and what is important.
I host free online workshop/Q&A sessions called Zen and the Art of LSAT. You can find upcoming dates here: http://www.manhattanlsat.com/zen-and-the-art.cfm
 
schwingrocker
Thanks Received: 0
Forum Guests
 
Posts: 23
Joined: July 01st, 2012
 
 
 

Re: Q17 - Consumer: If you buy a watch

by schwingrocker Sun Sep 02, 2012 7:40 pm

Is c wrong because "if the product does not perform" as it's expected is too broad?
 
alexg89
Thanks Received: 9
Jackie Chiles
Jackie Chiles
 
Posts: 39
Joined: July 24th, 2012
 
 
 

Re: Q17 - Consumer: If you buy a watch

by alexg89 Wed Nov 07, 2012 11:44 am

IMO the key to solving this is recognizing that BJ's does not have to be department store; it only says that by applying the principle laid out that they should refund them. However, for it to be supported they must have actually complied with the principle and the gap is D. (Analysis is nicely provided for that above)
 
dean.won
Thanks Received: 4
Forum Guests
 
Posts: 46
Joined: January 25th, 2013
 
 
 

Re: Q17 - Consumer: If you buy a watch

by dean.won Tue Jun 11, 2013 2:39 am

would C be a good principle answer?

and

would negating C be
"a seller should NOT refund the money if the product does perform as the purchaser expected it to perform" ?

thx
 
rumandredbull
Thanks Received: 0
Vinny Gambini
Vinny Gambini
 
Posts: 1
Joined: March 23rd, 2013
 
 
 

Re: Q17 - Consumer: If you buy a watch

by rumandredbull Fri Aug 09, 2013 8:48 am

Long time lurker, first time poster.

Really struggling to see why C is incorrect.
I thought D was incorrect because although it fills the gap about the watch being used as intended, wouldn't this be out of scope because we have established that BJ is not a department store?

Really confused, any help is appreciated.
 
nflamel69
Thanks Received: 16
Atticus Finch
Atticus Finch
 
Posts: 162
Joined: February 07th, 2011
 
 
 

Re: Q17 - Consumer: If you buy a watch

by nflamel69 Thu Sep 05, 2013 12:47 am

The key to this question is the conclusion. You have to recognize that its more or less a conditional conclusion. It says that So by reasonable standard, which suggest assuming the same principle as stated above, I needed to be refunded money. So what's the sufficient conditions that trigger the refund? 1) use it as it intends to be. 2) the watch broke next day. we are only given reason 2 in the premise. So we need the reason 1 to make the argument work.
User avatar
 
daniel
Thanks Received: 0
Elle Woods
Elle Woods
 
Posts: 62
Joined: July 31st, 2012
Location: Lancaster, CA
 
This post thanked 1 time.
 
 

Re: Q17 - Consumer: If you buy a watch

by daniel Sat Sep 07, 2013 8:57 pm

rumandredbull Wrote:Really struggling to see why C is incorrect.
.


I believe (C) can be eliminated because it is essentially a premise booster. That is, it restates the principle that is used as a premise of the argument, but extends it to include even more cases by eliminating one of the sufficient conditions (use the watch only as intended) and greatly expanding another ("stops working the next day" becomes "does not perform as expected").
User avatar
 
maryadkins
Thanks Received: 641
Atticus Finch
Atticus Finch
 
Posts: 1261
Joined: March 23rd, 2011
 
 
 

Re: Q17 - Consumer: If you buy a watch

by maryadkins Tue Sep 10, 2013 1:48 pm

Hey guys, great discussion here. To answer your questions:

daniel Wrote:I believe (C) can be eliminated because it is essentially a premise booster. That is, it restates the principle that is used as a premise of the argument, but extends it to include even more cases by eliminating one of the sufficient conditions (use the watch only as intended) and greatly expanding another ("stops working the next day" becomes "does not perform as expected").


Bingo. We're looking for a necessary assumption. First of all, (C) isn't so much an "assumption" because part of it is already stated. The part that's not stated isn't what we're looking for; it's too broad of a principle.

rumandredbull Wrote:I thought D was incorrect because although it fills the gap about the watch being used as intended, wouldn't this be out of scope because we have established that BJ is not a department store?


This is solved for by the phrase, "So by this very reasonable standard," which incorporates the department store standard to apply it to BJ's.

Like (C), (A) and (B) aren't in line with the principle given; they over-reach. (E) is out of scope (and why would the assumption be that it's NOT a new watch?)
 
nhahoyt
Thanks Received: 1
Vinny Gambini
Vinny Gambini
 
Posts: 5
Joined: January 23rd, 2017
 
 
 

Re: Q17 - Consumer: If you buy a watch

by nhahoyt Mon Jan 23, 2017 9:26 pm

I also chose B initially, but I see how D is correct. I'd like to add my takeaway to the thread:

As has been stated, it appears that there are two gaps in the consumer's logic. My initial answer was based off the gap relating to BJ's classification as a non-department store. Here's why I was wrong:

The consumer acknowledges that BJ's isn't a department store. Because of this explicit acknowledgment, it can no longer be called an assumption. An assumption is something that is by definition unstated.

That leaves only one remaining assumption: that the consumer used the watch as intended, which is stated in answer D.
 
jimmys143
Thanks Received: 0
Vinny Gambini
Vinny Gambini
 
Posts: 1
Joined: May 14th, 2018
 
 
 

Re: Q17 - Consumer: If you buy a watch

by jimmys143 Mon May 14, 2018 7:15 am

I will go with the option B and even i ask sellers here which sell patek phille watch to consider these things.
 
JeremyK460
Thanks Received: 0
Elle Woods
Elle Woods
 
Posts: 80
Joined: May 29th, 2020
 
 
 

Re: Q17 - Consumer: If you buy a watch

by JeremyK460 Fri Jul 23, 2021 6:34 pm

premise:
if you buy a watch at a DS and use it only how you're supposed to, but the watch breaks the next day, then you should be refunded

the watch broke the second i bought it from the jeweler

conclusion:
i should be refunded

one reason (amongst lots depending on your academic background) why even though the jeweler isn't a department store isn't necessary to the argument is because it's used in a concessive manner which expresses a non-truth conditional utterance. in other words...

even if it's raining, jeremy will go outside today.
jeremy will go outside today even though it's raining.

raining has no truth-conditional impact for this claim to be true

more particularly, even, in the way that the stimulus uses it, carries a conventional implicature (https://discovery.ucl.ac.uk/id/eprint/1 ... 324676.pdf) here's the link to the literature

it's a lot but just command/ctrl+F and search for key words. it's so dry, but happy readings!